LSAT and Law School Admissions Forum

Get expert LSAT preparation and law school admissions advice from PowerScore Test Preparation.

Questions relating to PowerScore's LSAT Logical Reasoning Bible
 abaskm
  • Posts: 7
  • Joined: Aug 14, 2020
|
#78068
Formal Logic Problem Set Answer Key - Question 1

“SP = counties with a single political party”

counties —> countries

It’s minor but thought i should let you know for the next edition.
User avatar
 Dave Killoran
PowerScore Staff
  • PowerScore Staff
  • Posts: 5850
  • Joined: Mar 25, 2011
|
#78086
Hi abaskm ,

Yes, I see that! Nice catch on your part, and I'll have that corrected for the next edition.

Much appreciated—thanks!
 abaskm
  • Posts: 7
  • Joined: Aug 14, 2020
|
#79955
Hi,

This is not an error but merely a suggestion:

In “Chapter 13 - Formal Logic Relationship Indicators Defined Numerically”.

You have the All/None listed in this order:

“All = 100

Most = 51 to 100 (“a majority”)

Some are not = 0 to 99 (also “Not All”)

Most are not = 0 to 49

Some = 1 to 100 (“at least one”)

None = 0”

Wouldn’t it make better sense to place Some right below most so that the All/None order reflects their respective ladders?

i.e.



“All = 100

Most = 51 to 100 (“a majority”)

Some = 1 to 100 (“at least one”)

Some are not = 0 to 99 (also “Not All”)

Most are not = 0 to 49

None = 0”

Excerpt From
The PowerScore LSAT Logical Reasoning Bible
David M. Killoran
https://books.apple.com/us/book/the-pow ... d912402390
This material may be protected by copyright.
 abaskm
  • Posts: 7
  • Joined: Aug 14, 2020
|
#80282
Hi Dave,

I think I may have found some more errors:

Chapter Twenty-One: Principle Questions - Principle Questions

“Which one of the following principles most helps to justify the reasoning above?” (Strengthen—PR) —> (Justify—PR)

“Which one of the following most accurately expresses the principle underlying the argumentation above?” (Justify—PR) —> (Must—PR) (Note: I could be wrong on this one, but I think it’s Must and not Strengthen because the principle is referenced in the stimulus not the answer choices.)

Chapter Twenty-Two: Section Strategy and Management - Guessing Strategy


“The test makers have plenty of tricks to keep you psychologically off-balance, and there are many odd answer choce grouping —> choice grouping(s?).”

Excerpt From
The PowerScore LSAT Logical Reasoning Bible
David M. Killoran
https://books.apple.com/us/book/the-pow ... d912402390
This material may be protected by copyright.
 Jeremy Press
PowerScore Staff
  • PowerScore Staff
  • Posts: 1000
  • Joined: Jun 12, 2017
|
#80382
Hi abaskm,

The question stem "Which one of the following principles most helps to justify the reasoning above?" is indeed a Strengthen-PR question. Just remember, the language "most justifies," or "most helps to justify," is always indicative of a Strengthen question, because it's setting a standard of justification that is potentially less than 100%. A Justify question is one in which the standard set in the question stem is (without qualification) 100%. For the stem you noted to be a Justify, it would need to read, "Which one of the following principles justifies the reasoning above?"

The second question stem can also technically be a Justify-PR question stem, though it's a little trickier to see why. Since the answer choice in that question will express "the principle underlying the argumentation" in the stimulus, assuming the argumentation is valid, the principle would Justify the Conclusion (in which case, Justify is a perfectly fine label). It may also be proper in certain circumstances to see that answer choice as an assumption (assumptions underlie arguments), and the question stem as an Assumption-PR. It would not be a Must Be True, though, because the principle wouldn't be something "following from" the stimulus.

The third one looks like a simple typo, and I'll flag that for folks.

Thanks again!
User avatar
 Dave Killoran
PowerScore Staff
  • PowerScore Staff
  • Posts: 5850
  • Joined: Mar 25, 2011
|
#80391
abaskm wrote:“The test makers have plenty of tricks to keep you psychologically off-balance, and there are many odd answer choce grouping —> choice grouping(s?).”

Excerpt From
The PowerScore LSAT Logical Reasoning Bible
David M. Killoran
https://books.apple.com/us/book/the-pow ... d912402390
This material may be protected by copyright.
Thanks, that has been fixed on iTunes!
 abaskm
  • Posts: 7
  • Joined: Aug 14, 2020
|
#80409
Jeremy Press wrote:Hi abaskm,

The question stem "Which one of the following principles most helps to justify the reasoning above?" is indeed a Strengthen-PR question. Just remember, the language "most justifies," or "most helps to justify," is always indicative of a Strengthen question, because it's setting a standard of justification that is potentially less than 100%. A Justify question is one in which the standard set in the question stem is (without qualification) 100%. For the stem you noted to be a Justify, it would need to read, "Which one of the following principles justifies the reasoning above?"

The second question stem can also technically be a Justify-PR question stem, though it's a little trickier to see why. Since the answer choice in that question will express "the principle underlying the argumentation" in the stimulus, assuming the argumentation is valid, the principle would Justify the Conclusion (in which case, Justify is a perfectly fine label). It may also be proper in certain circumstances to see that answer choice as an assumption (assumptions underlie arguments), and the question stem as an Assumption-PR. It would not be a Must Be True, though, because the principle wouldn't be something "following from" the stimulus.


The third one looks like a simple typo, and I'll flag that for folks.

Thanks again!
Thanks for the detailed answer Jeremy! It makes sense now. Also rereading the following sentence again:

“Which one of the following most accurately expresses the principle underlying the argumentation above?”

Does in fact imply that the principle resides in the answer choices and not in the stimulus.

It took me some time to wrap my head around it, but it makes sense now :-D

Get the most out of your LSAT Prep Plus subscription.

Analyze and track your performance with our Testing and Analytics Package.